Need Help Understanding Limits Question Algebraically

  • Thread starter Thread starter Manni
  • Start date Start date
  • Tags Tags
    Limits
Click For Summary
The discussion centers on understanding how to algebraically determine the coordinates of point Q as point P approaches the origin on a parabola defined by P=(x,x^2). The user seeks guidance on finding Q's y-coordinate, given that the x-coordinate is 0, and outlines steps involving geometry to find the midpoint, line equations, and perpendicular vectors. There is a clarification that Q does not approach infinity as P approaches the origin, contradicting the user's initial assumption. The conversation emphasizes the need for a clear algebraic expression for Q based on the defined geometric relationships. The focus remains on applying basic geometry principles to solve the problem effectively.
Manni
Messages
42
Reaction score
0
I need help with this question. I understand that logic behind it; as P approaches O the value the right bisector Q reaches it's maximum. I don't know how to show this algebraically however. Help?
 

Attachments

  • A5 Q6 .jpg
    A5 Q6 .jpg
    24.4 KB · Views: 496
Physics news on Phys.org
We give P the coordinates (x,y). We know that P is on the parabola, so we know that

P=(x,x^2)

Now, can you find the coordinates of the point Q??
 
Ok, so the bisector would intersect P at point x/2. Because I know that the x coordinate on Q is 0, how would I find it's y?
 
Answer the following steps:

- Find the midpoint M between 0 en P.
- Find the equation of the line L going through 0 and P
- Find a vector perpendicular to the line L
- Construct the equation of the line R going through M and perpendicular through L
- Find Q as the intersection between R and the y-axis.

All of these questions involve nothing more than 10th grade geometry. So you should be able to complete these easily.
 
How do I show algebraically that Q approaches infinity as P approaches the origin?
 
Manni said:
How do I show algebraically that Q approaches infinity as P approaches the origin?

It doesn't approach infinity.

Did you find the expression for Q?
 
Question: A clock's minute hand has length 4 and its hour hand has length 3. What is the distance between the tips at the moment when it is increasing most rapidly?(Putnam Exam Question) Answer: Making assumption that both the hands moves at constant angular velocities, the answer is ## \sqrt{7} .## But don't you think this assumption is somewhat doubtful and wrong?

Similar threads

  • · Replies 8 ·
Replies
8
Views
2K
  • · Replies 20 ·
Replies
20
Views
2K
Replies
5
Views
2K
  • · Replies 11 ·
Replies
11
Views
2K
Replies
34
Views
3K
Replies
2
Views
2K
  • · Replies 12 ·
Replies
12
Views
2K
  • · Replies 12 ·
Replies
12
Views
2K
  • · Replies 2 ·
Replies
2
Views
1K
  • · Replies 1 ·
Replies
1
Views
2K